Angular velocity of a loop after being struck by bullet

  • #1
brochesspro
155
22
Homework Statement
A circular hoop of mass m and radius R rests flat on a horizontal frictionless surface. A bullet, also of mass m and moving with velocity v, strikes the hoop and gets embedded in it. The thickness of the hoop is much smaller than R. The angular velocity with which the system rotates after the bullet strikes the hoop is:
Relevant Equations
Conservation of Angular Momentum
Parallel axis theorem to find Moment of Inertia
image.png

First off, I do know how to solve this problem. We use the principle of conservation of angular momentum about the centre of mass of the system which comprises of the loop and the bullet to obtain option B. My doubt is, why do we just not use the principle about the centre of the loop? Where is the problem in that?
 
Physics news on Phys.org
  • #2
brochesspro said:
Homework Statement: A circular hoop of mass m and radius R rests flat on a horizontal frictionless surface. A bullet, also of mass m and moving with velocity v, strikes the hoop and gets embedded in it. The thickness of the hoop is much smaller than R. The angular velocity with which the system rotates after the bullet strikes the hoop is:
Relevant Equations: Conservation of Angular Momentum
Parallel axis theorem to find Moment of Inertia

View attachment 337586
First off, I do know how to solve this problem. We use the principle of conservation of angular momentum about the centre of mass of the system which comprises of the loop and the bullet to obtain option B. My doubt is, why do we just not use the principle about the centre of the loop? Where is the problem in that?
In general, you can choose the centre of mass of the system or any fixed point in space.
 
  • #3
haruspex said:
In general, you can choose the centre of mass of the system or any fixed point in space.
The center of the hoop of course being neither of those.
 
  • #4
Orodruin said:
The center of the hoop of course being neither of those.
It was not clear to me how @brochesspro was defining that; could have meant the point where that is initially.
 
  • #5
haruspex said:
It was not clear to me how @brochesspro was defining that; could have meant the point where that is initially.
What do you mean? The y- co-ordinate of the CoM won't change, cuz there is no net force on the system, so all is fine, right? Since we only need the vertical distance from each individual object's centre of mass.
 
  • #6
haruspex said:
It was not clear to me how @brochesspro was defining that; could have meant the point where that is initially.
”Centre of the loop” to me sounds like the centre of the loop, not the CoM of the full system. Unless OP specifies with actual equations we can’t know for sure of course.

Edit: The OP also seems to make a distinction between the two …
 
  • #7
Orodruin said:
”Centre of the loop” to me sounds like the centre of the loop, not the CoM of the full system.
It does.
Orodruin said:
Unless OP specifies with actual equations we can’t know for sure of course.
Equations for what?
 
  • #8
brochesspro said:
Equations for what?
The equations you are actually writing down to model the evolution of the system. What we have so far is a verbal description
 
  • #9
Orodruin said:
The equations you are actually writing down to model the evolution of the system. What we have so far is a verbal description
Ok, so here it is.

We conserve angular momentum about the centre of mass of the loop, since all forces are internal, there are no external torques and hence this principle can be applied. The point where the bullet strikes the loop lies at a distance R from our reference point.

Initial angular momentum is given as ##MvR##. ie. before the bullet strikes the loop.

The moment of inertia of the system about the same point is given as ##mR^2 + mR^2 = 2mR^2##.

So, final angular momentum is given as ##mR^2 ω##.

Equating the two, we get ##ω## as ##ω = \frac v {2R}##.

Which does not match the actual answer of ##ω = \frac v {3R}##.

By the way, I have posted my approach in which I have a doubt, and not the correct one, I will post it if it is needed.
 
Last edited:
  • #10
brochesspro said:
This centre of mass lies at a distance ##R## from the centre between the centre of the loop and the point where the bullet strikes the loop.

Initial angular momentum is given as ##MvR##. ie. before the bullet strikes the loop.
What point are you taking as your reference for the angular momentum calculation?

Is that point moving?

To be clear, you can do the analysis about any reference you choose. That reference can be in any state of motion that you choose. But you need to choose. And you need to make that choice clear so that we can follow along and understand your work.

It appears that you have chosen the initial position of the center of mass of the loop as your reference and chosen a standard of rest in which the loop begins stationary.
 
  • #11
brochesspro said:
Ok, so here it is.

We conserve angular momentum about the centre of mass of the loop, since all forces are internal, there are no external torques and hence this principle can be applied. This centre of mass lies at a distance ##R## from the centre between the centre of the loop and the point where the bullet strikes the loop.

Initial angular momentum is given as ##MvR##. ie. before the bullet strikes the loop.

The moment of inertia of the system about the same point is given as ##mR^2 + mR^2 = 2mR^2##.

So, final angular momentum is given as ##mR^2 ω##.

Equating the two, we get ##ω## as ##ω = \frac v {2R}##.

Which does not match the actual answer of ##ω = \frac v {3R}##.

By the way, I have posted my approach in which I have a doubt, and not the correct one, I will post it if it is needed.
It's most concise to work around the combined systems center of mass as the reference for conservation of angular momentum here. The center of mass of the thin ring (alone - without the embedded bullet) is not the axis of rotation of the system after impact.
 
  • #12
erobz said:
The center of mass of the thin ring (alone) is not the axis of rotation of the system after impact.
How do we show this? And, can we use the same reference point and still get the correct answer? I have probably missed a few terms.
 
  • #13
brochesspro said:
How do we show this?
The center of mass of the system is where the instant of impact? The masses of the bullet ##m## and the ring ##m##.
brochesspro said:
And, can we use the same reference point and still get the correct answer? I have probably missed a few terms.
Yes, either way as long as you are careful about the accounting.
 
  • #14
brochesspro said:
We conserve angular momentum about the centre of mass of the loop, since all forces are internal, there are no external torques and hence this principle can be applied. This centre of mass lies at a distance R from the centre between the centre of the loop and the point where the bullet strikes the loop.
Your language here is extremely ambiguous.
The center of mass of the loop is some distance away from the center of mass of the loop? That distance is the same as the radius of the loop? I find it basically impossible to parse what you want to say.

brochesspro said:
Initial angular momentum is given as MvR. ie. before the bullet strikes the loop.
Now you are using R as the orthogonal distance from the center of the loop to where the bullet hits. This is compatible with using the center of the loop but not the center of mass of the system as the reference.

If you are not using the com as the reference, then you are missing terms due to the linear momentum of the system.
 
  • Like
Likes MatinSAR and jbriggs444
  • #15
Orodruin said:
The center of mass of the loop is some distance away from the center of mass of the loop? That distance is the same as the radius of the loop?
Sorry, I was thinking about the other method. let me correct it.
 
  • #16
brochesspro said:
Sorry, I was thinking about the other method. let me correct it.
I suggest trying to visualise the rotation of the loop and bullet after the collision. You can do this in the rest frame of the CoM, which must be moving inertially after the collision. Draw a few diagrams if necessary.

That also allows you to solve the problem by considering the rotation the bullet and the loop separately. That gives you a way to double-check your answer.
 
  • #17
jbriggs444 said:
What point are you taking as your reference for the angular momentum calculation?
I have corrected my sentence in my procedure, please check it.
erobz said:
The center of mass of the system is where the instant of impact?
I did not get this.
Orodruin said:
If you are not using the com as the reference, then you are missing terms due to the linear momentum of the system.
How do we obtain the expression for that?
 
  • #18
brochesspro said:
I did not get this.
I'm asking you to use the system center of mass as the reference.

1703343396241.png
 
  • #19
erobz said:
I'm asking you to use the system center of mass as the reference.

View attachment 337597
I did that and got the correct answer, but that is not my question. My question is how to get the correct answer by taking the reference as the centre of the loop.
 
  • Like
Likes erobz
  • #20
brochesspro said:
I did that and got the correct answer, but that is not my question. My question is how to get the correct answer by taking the reference as the centre of the loop.
The centre of the loop is accelerating after the collision. That makes things tricky.
 
  • Like
Likes erobz
  • #21
brochesspro said:
I did that and got the correct answer, but that is not my question. My question is how to get the correct answer by taking the reference as the centre of the loop.
I defer to the experts on that one. I always took it a face value that it can be computed about any point, but never tried to investigate how difficult that may be. I'm interested to see it too.
 
Last edited:
  • #22
Consider a rigid body with masses ##m_i## that have time dependent positions ##\vec x_i##. For such a body, the velocity field may be described by
$$
\vec v = \vec v_r + \vec \omega \times (\vec x - \vec x_r)
$$
where the ##r## subscript denotes a reference point. By definition, the angular momentum relative to the reference point is
$$
\vec L_r = \sum_i m_i (\vec x_i - \vec x_r) \times \vec v_i
= \sum_i m_i (\vec x_i - \vec x_r) \times(\vec v_r + \vec \omega \times (\vec x_i - \vec x_r)).
$$
We now use that ##\sum_i m_i \vec x_i = M \vec x_{cm}## defines the center of mass (with ##M## being the total mass):
$$
\vec L_r = M (\vec x_{cm} - \vec x_r) \times \vec v_r + I_r (\vec \omega)
$$
where ##I_r## is the moment of inertia tensor relative to ##\vec x_r##. The first term is zero in the the following cases:
  • ##\vec v_r = 0##: The reference point is chosen such that it has zero instantaneous velocity.
  • ##\vec x_r = \vec x_{cm}##: The reference point is the center of mass.
  • ##(\vec x_{cm} - \vec x_r)\times \vec v_r = 0##: The offset from the CoM to the reference point is parallel to ##\vec v_r##.
None of the above is applicable to the center of the hoop in your case so you cannot ignore the first term for the angular momentum after the collision.
 
  • Like
  • Informative
Likes nasu and PeroK
  • #23
Coming to the application to your problem:

You have two unknowns: The velocity ##\vec v_r## as well as the angular velocity ##\vec\omega##. To solve for those you will need both the conservation of angular momentum as well as the conservation of linear momentum. Using the CoM as reference point allows you to solve for ##\vec\omega## without involving the equation for linear momentum (because the first term above does not appear).

If you insist on using the center of the loop as reference, the linear momentum of the loop will be ##\vec v_r## and that of the bullet ##\vec v_r + \vec\omega \times(-R\vec e_y)## where the y-direction is chosen in the up direction in your figure. We can also assume ##\vec\omega = \omega\vec e_z## since angular velocity is restricted to be orthogonal to the horizontal plane. The linear momentum conservation becomes:
$$
mv \vec e_x = 2 m\vec v_r + m\omega R \vec e_x
$$
and the angular momentum conservation
$$
mvR \vec e_z = 2m (-R\vec e_y/2)\times \vec v_r + I_r \omega \vec e_z
$$
(assuming I didn’t do something whacky with the arithmetics - on mobile)

From the linear equation:
$$
\vec v_r = \vec e_x (v-\omega R)/2
$$
The angular equation therefore turns into
$$
mvR = mR(v-\omega R)/2 + 2mR^2 \omega
\quad
\Longrightarrow \quad
mvR/2 = 3 mR^2\omega /2
$$
and therefore
$$
\omega = v/3R
$$
 
Last edited:
  • Like
Likes erobz
  • #24
PeroK said:
The centre of the loop is accelerating after the collision. That makes things tricky.
Why do you say so? Did it not just gain velocity because of the impulse acting on it by the bullet? There is no other force during that time.
 
  • #25
brochesspro said:
Why do you say so? Did it not just gain velocity because of the impulse acting on it by the bullet? There is no other force during that time.
If you are saying the the hoop moves inertially, then that is wrong. That's why previously I advised you to:
PeroK said:
I suggest trying to visualise the rotation of the loop and bullet after the collision. You can do this in the rest frame of the CoM, which must be moving inertially after the collision. Draw a few diagrams if necessary.
 
  • #26
brochesspro said:
Why do you say so? Did it not just gain velocity because of the impulse acting on it by the bullet? There is no other force during that time.
The center of mass moves inertially. The center of the loop will move in a circular motion around the center of mass because there is angular velocity.
 
  • #27
brochesspro said:
What do you mean? The y- co-ordinate of the CoM won't change, cuz there is no net force on the system, so all is fine, right? Since we only need the vertical distance from each individual object's centre of mass.
I wasn't referring to the CoM.
You wrote that you want to use the centre of the hoop as the axis to take angular momentum about. You can do that if you mean the fixed point in space where the centre of the hoop is initially. It will give the wrong answer if you mean it in a dynamic sense, i.e. relative to where the centre of the hoop is and how it is moving at each instant.
Orodruin said:
”Centre of the loop” to me sounds like the centre of the loop, not the CoM of the full system.
See above.
 
Last edited:
  • #28
haruspex said:
You can do that if you mean the fixed point in space where the centre of the hoop is initially.
You can, but not in the way OP did it. (See #9)
You need an additional linear term apart from the moment of inertia term and you need to involve the conservation of linear momentum as well. (See #22 and #23)
 
  • #29
Orodruin said:
You can, but not in the way OP did it. (See #9)
Agreed
Orodruin said:
You need an additional linear term apart from the moment of inertia term
I'd put it differently. The angular momentum of the bullet about the initial hoop centre position is ##mv'r## where ##v'## is the new speed of the bullet in the ground frame. That equals ##mr^2\omega+mvr''##, where ##v''## is the linear speed of the hoop.
Post #9 omitted that second term.
Orodruin said:
and you need to involve the conservation of linear momentum as well. (See #22 and #23)
Sure, but I still see it as less effort than finding the CoM, as is often the case.
 
  • #30
brochesspro said:
Ok, so here it is.

We conserve angular momentum about the centre of mass of the loop, since all forces are internal, there are no external torques and hence this principle can be applied. The point where the bullet strikes the loop lies at a distance R from our reference point.

Initial angular momentum is given as ##MvR##. ie. before the bullet strikes the loop.

The moment of inertia of the system about the same point is given as ##mR^2 + mR^2 = 2mR^2##.

So, final angular momentum is given as ##mR^2 ω##.

Equating the two, we get ##ω## as ##ω = \frac v {2R}##.

Which does not match the actual answer of ##ω = \frac v {3R}##.

By the way, I have posted my approach in which I have a doubt, and not the correct one, I will post it if it is needed.
Why does the bullet initially have an R value? assuming that mvr=(12/4mr^2)ω
 
  • #31
Elj said:
Why does the bullet initially have an R value?
Do you mean, why does it have initial angular momentum mvR?
 
  • #32
Elj said:
Yes why does the bullet initially have angular momentum, if it is traveling linearly
Anything moving linearly on a path offset from the reference point has angular momentum.

Elj said:
assuming it is a point mass with some radius r,
This is nonsense. A point mass is a point and has no radius.

Elj said:
why does it cancel out with the radius of the loop
It … doesn’t?
 
  • #33
Orodruin said:
Anything moving linearly on a path offset from the reference point has angular momentum.This is nonsense. A point mass is a point and has no radius.It … doesn’t?
alright then can you explain how to solve it, Im genuinely confused
 
  • #34
Elj said:
alright then can you explain how to solve it, Im genuinely confused
I already did. See #22 and #23.
 
  • #35
Elj said:
alright then can you explain how to solve it, Im genuinely confused
Using conservation of angular momentum about the centre of mass, which is ##\frac R 2## below the centre of the hoop:
$$L = mv\frac R 2 = m(\frac R 2)^2 \omega + I \omega = (\frac{mR^2}{4} + I)\omega$$Where ##I## is the moment of inertia of the hoop about the centre of mass of the system. Using the parallel axis theorem we have:
$$I = mR^2 + m(\frac R 2)^2 = \frac{5mR^2}{4}$$This gives:
$$\frac{6mR^2}{4}\omega = mv\frac R 2$$$$\omega = \frac{v}{3R}$$
 

Similar threads

  • Introductory Physics Homework Help
2
Replies
50
Views
3K
  • Introductory Physics Homework Help
Replies
28
Views
550
  • Introductory Physics Homework Help
Replies
5
Views
1K
  • Introductory Physics Homework Help
2
Replies
45
Views
2K
  • Introductory Physics Homework Help
Replies
3
Views
1K
  • Introductory Physics Homework Help
Replies
6
Views
696
  • Introductory Physics Homework Help
Replies
10
Views
908
  • Introductory Physics Homework Help
10
Replies
335
Views
8K
  • Introductory Physics Homework Help
Replies
7
Views
1K
  • Introductory Physics Homework Help
Replies
30
Views
2K
Back
Top